Difference between revisions of "1985 AJHSME Problem 2"

(Created page with "==Problem== <math>90+91+92+93+94+95+96+97+98+99=</math> <math>\text{(A)}\ 845 \qquad \text{(B)}\ 945 \qquad \text{(C)}\ 1005 \qquad \text{(D)}\ 1025 \qquad \text{(E)}\ 1045<...")
 
 
Line 17: Line 17:
 
==Cheap Solution==
 
==Cheap Solution==
 
We know that <math>10(90) = 900</math> and <math>10(100) = 1000.</math> Quick estimation reveals that this sum is in between these two numbers, so the only answer available is <math>\text{(B)}.</math>
 
We know that <math>10(90) = 900</math> and <math>10(100) = 1000.</math> Quick estimation reveals that this sum is in between these two numbers, so the only answer available is <math>\text{(B)}.</math>
 +
 +
==Video Solution==
 +
https://youtu.be/1NtsgKc6mXs
 +
 +
~savannahsolver

Latest revision as of 08:36, 7 January 2023

Problem

$90+91+92+93+94+95+96+97+98+99=$


$\text{(A)}\ 845 \qquad \text{(B)}\ 945 \qquad \text{(C)}\ 1005 \qquad \text{(D)}\ 1025 \qquad \text{(E)}\ 1045$

Solution 1

We can add as follows: \[90+91+92+93+94+95+96+97+98+99= 10(90) +1+2+3+4+5+6+7+8+9 = 900 + 45 = \boxed{945}\] The answer is $\text{(B)}.$

Solution 2

Pair the numbers like so: \[(90+99)+(91+98)+(92+97)+(93+96)+(94+95)\] The sum of each pair is $189$ and there are $5$ pairs, so the sum is $945$ and the answer is $\text{(B)}.$

Cheap Solution

We know that $10(90) = 900$ and $10(100) = 1000.$ Quick estimation reveals that this sum is in between these two numbers, so the only answer available is $\text{(B)}.$

Video Solution

https://youtu.be/1NtsgKc6mXs

~savannahsolver